0



Wenn ich das richtig verstanden habe, berechne ich hier ein Wegintegral 1. Art, . Bedeutet mein Vorgehen wäre es in Kreiskoordinaten zu parametrisieren.
Am Ende habe ich das raus? Hab sozusagen doch nur den Viertelkreis berechnet. Der Weg/ die Gerade -x+3 im Kreis gar nicht betrachtet? Also meine frage ist, wie gehe ich mit den Punkten um, wie eine gerade im Kreis ist und die andere sogar außerhalb. (muss diese überhaupt berücksichtigt werden ?)


EDIT vom 19.08.2021 um 16:38:

Kann keine Kommentare schreiben, noch etwas bewerten oder weiteres... Bearbeite jetzt diese Frage. Zu meinen integralen mit der Kreis Parametrisierung. Wie schon Festgestell ist dieses ein Arbeitsintegral, welches null ergibt. Hatte das gerade nochmal Nachgerechnet und dennoch habe ich -9+9 = 0 raus. Mein Integral sieht wie folgt aus: entstanden wie unten schon in den Kommentaren beschrieben. \(\int_0^{\frac{pi}{2}} 9\cost^2*(-3\sint)+9\sint^2*3\cost dt\) \([9\cost^3] + [9\sint^3] von 0 bis pi/2 \)
Diese Frage melden
gefragt

Punkte: 27

 
Kommentar schreiben
1 Antwort
1
Zunächst muss die Kurve richtig aufgestellt werden. Diese besteht aus zwei Teilen, ein Viertelkreis, sagen wir $\gamma_1$, und eine Strecke, sagen wir $\gamma_2$. Zu der richtigen Parametrisierung gehört das richtige Intervall (Probe: Läuft die Kurve von ... nach..., wie sie laut Aufgabenstellung soll?). Dein Viertelkreis tut das nicht, weil das Intervall fehlt und der Radius nicht stimmt. Heißt, Du hast keine Probe gemacht. Tu das unbedingt.
Dann fehlt noch der zweite Teil, $\gamma_2$, Regeln dafür s.o. (Probe!).
Das gesuchte Integral ist dann einfach $\int_{\gamma_1}+\int_{\gamma_2}$.
Diese Antwort melden
geantwortet

Lehrer/Professor, Punkte: 38.86K

 

Tut mir leid, ich hatte es oben falsch eingegeben. Die Grenzen sollten von 0 bis pi/2 laufen, da meine eine gerade von den punkten (3,0) und (0,3) läuft. Sehe auch das die Ableitung falsch ist.

Also die richtige Parametrisierung mit dem Radius einbezogen wäre:
γ1= (rcos(t), rsin(t))
γ'1= (-rsin(t), rcos(t))
|γ1| = r

Doppelintegral mit den Grenzen
0<= t <= pi/2
0<= r <= 3

im Integral steht dann:

S((rcos(t))^2+(rsin(t))^2)*r drdt = 81pi/4

Das wäre der erste Teil.





  ─   user383f24 04.08.2021 um 16:04

Okay hab mich wieder ran gesetzt und glaube jetzt hab ich es.
Für den ersten Teil meine Parametrisierung ist
γ1 = (3cos(t),3sin(t)) 0<=t<=pi/2 -> probe: für t= 0 (3,0); für t = p/2 (0,3)
γ'1 = (-3sin(t),3cos(t))

Dann eingesetzt ergeben sich später zwei einfache Integrale : -9+9 =0

für den zweiten Teil geht es um die Gerade mit den Punkte (0,3) und (5,6)

γ2 = (5/3t, t+3) 0 <= t <= 3
γ'2 = (5/3, 1)

Eingesetzt erhalte ich 314/3 ?

Bedeutet 0+314/3 = 314/3
  ─   user383f24 10.08.2021 um 12:36

Leider scheint diese Antwort Unstimmigkeiten zu enthalten und muss korrigiert werden. Mikn wurde bereits informiert.